Recent content by Wormaldson

  1. W

    Limit problem involving x to the power of a function

    Homework Statement Calculate the limit \lim_{x\rightarrow +\infty} x^{\frac{1}{x^2}} 2. The attempt at a solution At first I was tempted to rewrite the function as a composition and go from there. I soon realized that this wouldn't work as the term being raised to the power of a function of...
  2. W

    Question on relationships between nuclear models

    Hm. That's very interesting. Is there any particular reason why it still works besides being unreasonable? And as a follow-up question, are there more accurate models currently in use?
  3. W

    Question on relationships between nuclear models

    So far in my physics education I've developed a basic understanding of two nuclear models, the liquid-drop model and the shell model. I read something a while ago (don't have the text on hand to quote the exact phrasing, unfortunately) that seemed to imply, in a couple of places, at least...
  4. W

    Simple Pulley System with Two Masses

    Huh. Well then. Thanks for the confirmation.
  5. W

    Simple Pulley System with Two Masses

    Homework Statement http://imgur.com/Y1Dua r = 0.2m I = 1.4kg m^2 m1 = 2kg m2 = 5kg h = 4m Homework Equations Δ(Gravitational potential energy) = m1*g*h - m2*g*h Δ(Kinetic energy) = (1/2)*m1*v^2 + (1/2)*m2*v^2 + (1/2)*I*(v^2/r^2) The Attempt at a Solution System is...
  6. W

    Question on the probabilistic nature of QM

    Question on the "probabilistic" nature of QM I read very recently something that I interpreted as stating that certain quantum-mechanical phenomena are necessarily probability-based: for instance the exact path traversed by a photon/electron in the double-slit experiment. That's all well and...
  7. W

    Apparent Discrepancy Between Two Definitions of Newton's First Law

    Thanks everyone for the replies, unfortunately I'm still unsure as to how exactly the First Law is supposed to be interpreted. Is there no universal general consensus on the matter? And to supplement the information given I think I should include the entire quote from the textbook (I didn't...
  8. W

    Apparent Discrepancy Between Two Definitions of Newton's First Law

    Thanks for the reply. Based on what you've said, I gather that I should: assume that the example from the textbook was indeed implicitly making the assumption that we should only consider forces acting on the hockey puck horizontally, and assume that the Wikipedia article should not be...
  9. W

    Apparent Discrepancy Between Two Definitions of Newton's First Law

    So I was reading through my textbook (specifically, Physics for Scientists and Engineers, Eighth Edition, Volume 1 by Raymond A. Serway and John W. Jewett Jr.) and I noticed that, in one of the "Pitfall Prevention" sections (which are usually quite helpful - not this time, evidently), it says...
  10. W

    Simple Differentiability and Continuity Question

    You're right; I was considering the problem in too narrow a scope and only considering the limit as x\rightarrow0. So let's see: the limit as x\rightarrow0 of f(x) does not exist, therefore, the limit as h\rightarrow0 of f(0 + h) - f(0) does not exist either. The limit as h\rightarrow0 of h is...
  11. W

    Simple Differentiability and Continuity Question

    Homework Statement If f(x) = 3 for x < 0 and f(x) = 2x for x ≥ 0, is f(x) differentiable at x = 0? State and justify why/why not. Homework Equations The Attempt at a Solution Obviously, since f(x) is not continuous and the limit doesn't exist as x\rightarrow0, the function...
  12. W

    Confusing Error in Should-Be-Simple Integration Problem

    Righto, found out what I was doing wrong: I somehow forgot that logs don't distribute over multiplication. How silly of me. k = 5*ln(5 + e^(1/5)) Edit: Thanks for the help.
  13. W

    Confusing Error in Should-Be-Simple Integration Problem

    Homework Statement Find k such that the area between the function and the x-axis, bounded by x = 1 and x = k, is equal to 100 square units. Homework Equations y = 4e^(x/5) The Attempt at a Solution 1. Wrote the problem as the definite integral from 1 to k of 4e^(x/5)dx = 100...
  14. W

    (Another) Introductory Level Calculus Question

    Thank you sir, I think I've got it now. Will update with the working when I get the time to do it. I'm aware of that, but I don't have much of a choice, really. My exams are coming up fast, and I'm aiming to score well in them, so I have to sort of "force-feed" myself this stuff as quickly...
  15. W

    (Another) Introductory Level Calculus Question

    Thanks, that's a step in the right direction.
Back
Top